Are you seeking one-on-one college counseling and/or essay support? Limited spots are now available. Click here to learn more.

How to Write the AP Lang Rhetorical Analysis Essay (With Example)

November 27, 2023

Feeling intimidated by the AP Lang Rhetorical Analysis Essay? We’re here to help demystify. Whether you’re cramming for the AP Lang exam right now or planning to take the test down the road, we’ve got crucial rubric information, helpful tips, and an essay example to prepare you for the big day. This post will cover 1) What is the AP Lang Rhetorical Analysis Essay? 2) AP Lang Rhetorical Analysis Rubric 3) AP Lang Rhetorical Analysis: Sample Prompt 4) AP Lang Rhetorical Analysis Essay Example 5)AP Lang Rhetorical Analysis Essay Example: Why It Works

What is the AP Lang Rhetorical Analysis Essay?

The AP Lang Rhetorical Analysis Essay is one of three essays included in the written portion of the AP English Exam. The full AP English Exam is 3 hours and 15 minutes long, with the first 60 minutes dedicated to multiple-choice questions. Once you complete the multiple-choice section, you move on to three equally weighted essays that ask you to synthesize, analyze, and interpret texts and develop well-reasoned arguments. The three essays include:

Synthesis essay: You’ll review various pieces of evidence and then write an essay that synthesizes (aka combines and interprets) the evidence and presents a clear argument. Read our write up on How to Write the AP Lang Synthesis Essay here.

Argumentative essay: You’ll take a stance on a specific topic and argue your case.

Rhetorical essay: You’ll read a provided passage, then analyze the author’s rhetorical choices and develop an argument that explains why the author made those rhetorical choices.

AP Lang Rhetorical Analysis Rubric

The AP Lang Rhetorical Analysis Essay is graded on just 3 rubric categories: Thesis, Evidence and Commentary, and Sophistication . At a glance, the rubric categories may seem vague, but AP exam graders are actually looking for very particular things in each category. We’ll break it down with dos and don’ts for each rubric category:

Thesis (0-1 point)

There’s nothing nebulous when it comes to grading AP Lang Rhetorical Analysis Essay thesis. You either have one or you don’t. Including a thesis gets you one point closer to a high score and leaving it out means you miss out on one crucial point. So, what makes a thesis that counts?

  • Make sure your thesis argues something about the author’s rhetorical choices. Making an argument means taking a risk and offering your own interpretation of the provided text. This is an argument that someone else might disagree with.
  • A good test to see if you have a thesis that makes an argument. In your head, add the phrase “I think that…” to the beginning of your thesis. If what follows doesn’t logically flow after that phrase (aka if what follows isn’t something you and only you think), it’s likely you’re not making an argument.
  • Avoid a thesis that merely restates the prompt.
  • Avoid a thesis that summarizes the text but does not make an argument.

Evidence and Commentary (0-4 points)

This rubric category is graded on a scale of 0-4 where 4 is the highest grade. Per the AP Lang Rhetorical Analysis rubric, to get a 4, you’ll want to:

  • Include lots of specific evidence from the text. There is no set golden number of quotes to include, but you’ll want to make sure you’re incorporating more than a couple pieces of evidence that support your argument about the author’s rhetorical choices.
  • Make sure you include more than one type of evidence, too. Let’s say you’re working on your essay and have gathered examples of alliteration to include as supporting evidence. That’s just one type of rhetorical choice, and it’s hard to make a credible argument if you’re only looking at one type of evidence. To fix that issue, reread the text again looking for patterns in word choice and syntax, meaningful figurative language and imagery, literary devices, and other rhetorical choices, looking for additional types of evidence to support your argument.
  • After you include evidence, offer your own interpretation and explain how this evidence proves the point you make in your thesis.
  • Don’t summarize or speak generally about the author and the text. Everything you write must be backed up with evidence.
  • Don’t let quotes speak for themselves. After every piece of evidence you include, make sure to explain your interpretation. Also, connect the evidence to your overarching argument.

Sophistication (0-1 point)

In this case, sophistication isn’t about how many fancy vocabulary words or how many semicolons you use. According to College Board , one point can be awarded to AP Lang Rhetorical Analysis essays that “demonstrate sophistication of thought and/or a complex understanding of the rhetorical situation” in any of these three ways:

  • Explaining the significance or relevance of the writer’s rhetorical choices.
  • Explaining the purpose or function of the passage’s complexities or tensions.
  • Employing a style that is consistently vivid and persuasive.

Note that you don’t have to achieve all three to earn your sophistication point. A good way to think of this rubric category is to consider it a bonus point that you can earn for going above and beyond in depth of analysis or by writing an especially persuasive, clear, and well-structured essay. In order to earn this point, you’ll need to first do a good job with your thesis, evidence, and commentary.

  • Focus on nailing an argumentative thesis and multiple types of evidence. Getting these fundamentals of your essay right will set you up for achieving depth of analysis.
  • Explain how each piece of evidence connects to your thesis.
  • Spend a minute outlining your essay before you begin to ensure your essay flows in a clear and cohesive way.
  • Steer clear of generalizations about the author or text.
  • Don’t include arguments you can’t prove with evidence from the text.
  • Avoid complex sentences and fancy vocabulary words unless you use them often. Long, clunky sentences with imprecisely used words are hard to follow.

AP Lang Rhetorical Analysis: Sample Prompt

The sample prompt below is published online by College Board and is a real example from the 2021 AP Exam. The prompt provides background context, essay instructions, and the text you need to analyze. For sake of space, we’ve included the text as an image you can click to read. After the prompt, we provide a sample high scoring essay and then explain why this AP Lang Rhetorical Analysis essay example works.

Suggested time—40 minutes.

(This question counts as one-third of the total essay section score.)

On February 27, 2013, while in office, former president Barack Obama delivered the following address dedicating the Rosa Parks statue in the National Statuary Hall of the United States Capitol building. Rosa Parks was an African American civil rights activist who was arrested in 1955 for refusing to give up her seat on a segregated bus in Montgomery, Alabama. Read the passage carefully. Write an essay that analyzes the rhetorical choices Obama makes to convey his message.

In your response you should do the following:

  • Respond to the prompt with a thesis that analyzes the writer’s rhetorical choices.
  • Select and use evidence to support your line of reasoning.
  • Explain how the evidence supports your line of reasoning.
  • Demonstrate an understanding of the rhetorical situation.
  • Use appropriate grammar and punctuation in communicating your argument.

AP Lang Rhetorical Analysis Essay Example

In his speech delivered in 2013 at the dedication of Rosa Park’s statue, President Barack Obama acknowledges everything that Parks’ activism made possible in the United States. Telling the story of Parks’ life and achievements, Obama highlights the fact that Parks was a regular person whose actions accomplished enormous change during the civil rights era. Through the use of diction that portrays Parks as quiet and demure, long lists that emphasize the extent of her impacts, and Biblical references, Obama suggests that all of us are capable of achieving greater good, just as Parks did.

Although it might be a surprising way to start to his dedication, Obama begins his speech by telling us who Parks was not: “Rosa Parks held no elected office. She possessed no fortune” he explains in lines 1-2. Later, when he tells the story of the bus driver who threatened to have Parks arrested when she refused to get off the bus, he explains that Parks “simply replied, ‘You may do that’” (lines 22-23). Right away, he establishes that Parks was a regular person who did not hold a seat of power. Her protest on the bus was not part of a larger plan, it was a simple response. By emphasizing that Parks was not powerful, wealthy, or loud spoken, he implies that Parks’ style of activism is an everyday practice that all of us can aspire to.

AP Lang Rhetorical Analysis Essay Example (Continued)

Even though Obama portrays Parks as a demure person whose protest came “simply” and naturally, he shows the importance of her activism through long lists of ripple effects. When Parks challenged her arrest, Obama explains, Martin Luther King, Jr. stood with her and “so did thousands of Montgomery, Alabama commuters” (lines 27-28). They began a boycott that included “teachers and laborers, clergy and domestics, through rain and cold and sweltering heat, day after day, week after week, month after month, walking miles if they had to…” (lines 28-31). In this section of the speech, Obama’s sentences grow longer and he uses lists to show that Parks’ small action impacted and inspired many others to fight for change. Further, listing out how many days, weeks, and months the boycott lasted shows how Parks’ single act of protest sparked a much longer push for change.

To further illustrate Parks’ impact, Obama incorporates Biblical references that emphasize the importance of “that single moment on the bus” (lines 57-58). In lines 33-35, Obama explains that Parks and the other protestors are “driven by a solemn determination to affirm their God-given dignity” and he also compares their victory to the fall the “ancient walls of Jericho” (line 43). By of including these Biblical references, Obama suggests that Parks’ action on the bus did more than correct personal or political wrongs; it also corrected moral and spiritual wrongs. Although Parks had no political power or fortune, she was able to restore a moral balance in our world.

Toward the end of the speech, Obama states that change happens “not mainly through the exploits of the famous and the powerful, but through the countless acts of often anonymous courage and kindness” (lines 78-81). Through carefully chosen diction that portrays her as a quiet, regular person and through lists and Biblical references that highlight the huge impacts of her action, Obama illustrates exactly this point. He wants us to see that, just like Parks, the small and meek can change the world for the better.

AP Lang Rhetorical Analysis Essay Example: Why It Works

We would give the AP Lang Rhetorical Analysis essay above a score of 6 out of 6 because it fully satisfies the essay’s 3 rubric categories: Thesis, Evidence and Commentary, and Sophistication . Let’s break down what this student did:

The thesis of this essay appears in the last line of the first paragraph:

“ Through the use of diction that portrays Parks as quiet and demure, long lists that emphasize the extent of her impacts, and Biblical references, Obama suggests that all of us are capable of achieving greater good, just as Parks did .”

This student’s thesis works because they make a clear argument about Obama’s rhetorical choices. They 1) list the rhetorical choices that will be analyzed in the rest of the essay (the italicized text above) and 2) include an argument someone else might disagree with (the bolded text above).

Evidence and Commentary:

This student includes substantial evidence and commentary. Things they do right, per the AP Lang Rhetorical Analysis rubric:

  • They include lots of specific evidence from the text in the form of quotes.
  • They incorporate 3 different types of evidence (diction, long lists, Biblical references).
  • After including evidence, they offer an interpretation of what the evidence means and explain how the evidence contributes to their overarching argument (aka their thesis).

Sophistication

This essay achieves sophistication according to the AP Lang Rhetorical Analysis essay rubric in a few key ways:

  • This student provides an introduction that flows naturally into the topic their essay will discuss. Before they get to their thesis, they tell us that Obama portrays Parks as a “regular person” setting up their main argument: Obama wants all regular people to aspire to do good in the world just as Rosa Parks did.
  • They organize evidence and commentary in a clear and cohesive way. Each body paragraph focuses on just one type of evidence.
  • They explain how their evidence is significant. In the final sentence of each body paragraph, they draw a connection back to the overarching argument presented in the thesis.
  • All their evidence supports the argument presented in their thesis. There is no extraneous evidence or misleading detail.
  • They consider nuances in the text. Rather than taking the text at face value, they consider what Obama’s rhetorical choices imply and offer their own unique interpretation of those implications.
  • In their final paragraph, they come full circle, reiterate their thesis, and explain what Obama’s rhetorical choices communicate to readers.
  • Their sentences are clear and easy to read. There are no grammar errors or misused words.

AP Lang Rhetorical Analysis Essay—More Resources

Looking for more tips to help your master your AP Lang Rhetorical Analysis Essay? Brush up on 20 Rhetorical Devices High School Students Should Know and read our Tips for Improving Reading Comprehension . If you’re ready to start studying for another part of the AP English Exam, find more expert tips in our How to Write the AP Lang Synthesis blog post.

Considering what other AP classes to take? Read up on the Hardest AP Classes .

  • High School Success

Christina Wood

Christina Wood holds a BA in Literature & Writing from UC San Diego, an MFA in Creative Writing from Washington University in St. Louis, and is currently a Doctoral Candidate in English at the University of Georgia, where she teaches creative writing and first-year composition courses. Christina has published fiction and nonfiction in numerous publications, including The Paris Review , McSweeney’s , Granta , Virginia Quarterly Review , The Sewanee Review , Mississippi Review , and Puerto del Sol , among others. Her story “The Astronaut” won the 2018 Shirley Jackson Award for short fiction and received a “Distinguished Stories” mention in the 2019 Best American Short Stories anthology.

  • 2-Year Colleges
  • ADHD/LD/Autism/Executive Functioning
  • Application Strategies
  • Best Colleges by Major
  • Best Colleges by State
  • Big Picture
  • Career & Personality Assessment
  • College Essay
  • College Search/Knowledge
  • College Success
  • Costs & Financial Aid
  • Data Visualizations
  • Dental School Admissions
  • Extracurricular Activities
  • General Knowledge
  • Graduate School Admissions
  • High Schools
  • Homeschool Resources
  • Law School Admissions
  • Medical School Admissions
  • Navigating the Admissions Process
  • Online Learning
  • Outdoor Adventure
  • Private High School Spotlight
  • Research Programs
  • Summer Program Spotlight
  • Summer Programs
  • Teacher Tools
  • Test Prep Provider Spotlight

“Innovative and invaluable…use this book as your college lifeline.”

— Lynn O'Shaughnessy

Nationally Recognized College Expert

College Planning in Your Inbox

Join our information-packed monthly newsletter.

AP Research

Learn all about the course and assessment. Already enrolled? Join your class in My AP.

Not a Student?

Go to AP Central for resources for teachers, administrators, and coordinators.

About the Assessment

There is no end-of-course written exam for AP Research. Instead, you’ll be assessed on performance tasks you complete that are based on your yearlong research project: an academic paper (which you’ll submit online for scoring through the AP Digital Portfolio), a presentation, and an oral defense of your research. These components all contribute to your final AP score on a scale of 1–5.

Updates to AP Research Performance Task

Given the implications of ChatGPT and other similar generative artificial intelligence (AI) tools on the AP Research performance task components, we’ve updated the guidance  regarding the use of these tools. These guidelines require your teacher to attest to the authenticity of your work in order for you to receive a score on the associated performance task. Review the  AP Research Course and Exam Description for the full policy and details.

Assessment Date

Wed, Apr 30, 2025

11:59 PM ET

AP Research Performance Task Due Date

Submit your AP Research performance task as final in the AP Digital Portfolio by this date.

Assessment Components

Academic paper.

75% of Score

The academic paper should be 4,000–5,000 words long. You’ll be evaluated on the content, structure, format, and conclusions of the paper as well as your ability to properly and accurately cite sources.

Presentation and Oral Defense

25% of Score

The culminating event of the AP Research course will be a presentation of your research question, research methodology, and findings, including an oral defense that addresses a set of questions about your research inquiry. The presentation and defense take 15–20 minutes. You will also be required to answer 3–4 questions from a panel of trained evaluators and your AP Research teacher.

  • AP Research teachers use a scoring rubric designed by the AP Program.
  • AP Research teachers also take part in mandatory training from the AP Program in how to score these components.

How We Score Your Work

Teacher Scored Components

Your in-class presentation and oral defense is scored by your AP Research teacher.

Here’s how we make sure that scoring by AP Research teachers is standardized:

College Board Scored Components

After you submit your academic paper online through the AP Digital Portfolio, it is scored by trained, experienced educators called AP readers.

Assessment Essentials

Assessment preparation, ap daily videos.

Once you join your AP class section online, you’ll be able to access AP Daily videos in AP Classroom. AP Daily videos cover every proficiency and skill outlined in the AP Research Course and Exam Description. Sign in to access them.

Performance Task Samples and Scoring Information

Go to AP Central to review student samples of the academic paper, as well as scoring information for both performance tasks, from past years.

AP Research Course and Exam Description

This is the core document for the course. It clearly lays out the course content and describes the assessment and the AP Program in general.

Services for Students with Disabilities

Students with documented disabilities may be eligible for accommodations for the through-course assessment and the end-of-course exam. If you’re using assistive technology and need help accessing the PDFs in this section in another format, contact Services for Students with Disabilities at 212-713-8333 or by email at [email protected] . For information about taking AP Exams, or other College Board assessments, with accommodations, visit the Services for Students with Disabilities website.

Guidance for Artificial Intelligence Tools and Other Services

Learn more about the AP Program’s policy on generative AI tools (e.g., ChatGPT).

Credit and Placement

Search AP Credit Policies

Find colleges that grant credit and/or placement for AP Exam scores in this and other AP courses.

Additional Information

What are your chances of acceptance?

Calculate for all schools, your chance of acceptance.

Duke University

Your chancing factors

Extracurriculars.

ap english research paper example

How to Write the AP Lang Argument Essay + Examples

What’s covered:, what is the ap language argument essay, tips for writing the ap language argument essay, ap english language argument essay examples, how will ap scores impact my college chances.

In 2023, over 550,148 students across the U.S. took the AP English Language and Composition Exam, and 65.2% scored higher than a 3. The AP English Language Exam tests your ability to analyze a piece of writing, synthesize information, write a rhetorical essay, and create a cohesive argument. In this post, we’ll be discussing the best way to approach the argumentative essay section of the test, and we’ll give you tips and tricks so you can write a great essay.

The AP English Language Exam as of 2023 is structured as follows:

Section 1: 45 multiple choice questions to be completed in an hour. This portion counts for 45% of your score. This section requires students to analyze a piece of literature. The questions ask about its content and/or what could be edited within the passage.

Section 2: Three free response questions to be completed in the remaining two hours and 15 minutes. This section counts for 55% of your score. These essay questions include the synthesis essay, the rhetorical essay, and the argumentative essay.

  • Synthesis essay: Read 6-7 sources and create an argument using at least three of the sources.
  • Rhetorical analysis essay: Describe how a piece of writing evokes meaning and symbolism.
  • Argumentative essay: Pick a side of a debate and create an argument based on evidence. In this essay, you should develop a logical argument in support of or against the given statement and provide ample evidence that supports your conclusion. Typically, a five paragraph format is great for this type of writing. This essay is scored holistically from 1 to 9 points.

Do you want more information on the structure of the full exam? Take a look at our in-depth overview of the AP Language and Composition Exam .

Although the AP Language Argument may seem daunting at first, once you understand how the essay should be structured, it will be a lot easier to create cohesive arguments.

Below are some tips to help you as you write the essay.

1. Organize your essay before writing

Instead of jumping right into your essay, plan out what you will say beforehand. It’s easiest to make a list of your arguments and write out what facts or evidence you will use to support each argument. In your outline, you can determine the best order for your arguments, especially if they build on each other or are chronological. Having a well-organized essay is crucial for success.

2. Pick one side of the argument, but acknowledge the other side

When you write the essay, it’s best if you pick one side of the debate and stick with it for the entire essay. All your evidence should be in support of that one side. However, in your introductory paragraph, as you introduce the debate, be sure to mention any merit the arguments of the other side has. This can make the essay a bit more nuanced and show that you did consider both sides before determining which one was better. Often, acknowledging another viewpoint then refuting it can make your essay stronger.

3. Provide evidence to support your claims

The AP readers will be looking for examples and evidence to support your argument. This doesn’t mean that you need to memorize a bunch of random facts before the exam. This just means that you should be able to provide concrete examples in support of your argument.

For example, if the essay topic is about whether the role of the media in society has been detrimental or not, and you argue that it has been, you may talk about the phenomenon of “fake news” during the 2016 presidential election.

AP readers are not looking for perfect examples, but they are looking to see if you can provide enough evidence to back your claim and make it easily understood.

4. Create a strong thesis statement

The thesis statement will set up your entire essay, so it’s important that it is focused and specific, and that it allows for the reader to understand your body paragraphs. Make sure your thesis statement is the very last sentence of your introductory paragraph. In this sentence, list out the key points you will be making in the essay in the same order that you will be writing them. Each new point you mention in your thesis should start a paragraph in your essay.

Below is a prompt and sample student essay from the May 2019 exam . We’ll look at what the student did well in their writing and where they could improve.

Prompt: “The term “overrated” is often used to diminish concepts, places, roles, etc. that the speaker believes do not deserve the prestige they commonly enjoy; for example, many writers have argued that success is overrated, a character in a novel by Anthony Burgess famously describes Rome as a “vastly overrated city,” and Queen Rania of Jordan herself has asserted that “[b]eing queen is overrated.”

Select a concept, place, role, etc. to which you believe that the term “overrated” should be applied. Then, write a well-developed essay in which you explain your judgment. Use appropriate evidence from your reading, experience, or observations to support your argument.

Sample Student Essay #1:

[1] Competition is “overrated.” The notion of motivation between peers has evolved into a source of unnecessary stress and even lack of morals. Whether it be in an academic environment or in the industry, this new idea of competition is harmful to those competing and those around them.

[2] Back in elementary school, competition was rather friendly. It could have been who could do the most pushups or who could get the most imaginary points in a classroom for a prize. If you couldn’t do the most pushups or win that smelly sticker, you would go home and improve yourself – there would be no strong feelings towards anyone, you would just focus on making yourself a better version of yourself. Then as high school rolled around, suddenly applying for college doesn’t seem so far away –GPA seems to be that one stat that defines you – extracurriculars seem to shape you – test scores seem to categorize you. Sleepless nights, studying for the next day’s exam, seem to become more and more frequent. Floating duck syndrome seems to surround you (FDS is where a competitive student pretends to not work hard but is furiously studying beneath the surface just like how a duck furiously kicks to stay afloat). All of your competitors appear to hope you fail – but in the end what do you and your competitor’s gain? Getting one extra point on the test? Does that self-satisfaction compensate for the tremendous amounts of acquired stress? This new type of “competition” is overrated – it serves nothing except a never-ending source of anxiety and seeks to weaken friendships and solidarity as a whole in the school setting.

[3] A similar idea of “competition” can be applied to business. On the most fundamental level, competition serves to be a beneficial regulator of prices and business models for both the business themselves and consumers. However, as businesses grew increasingly greedy and desperate, companies resorted to immoral tactics that only hurt their reputations and consumers as a whole. Whether it be McDonald’s coupons that force you to buy more food or tech companies like Apple intentionally slowing down your iPhone after 3 years to force you to upgrade to the newest device, consumers suffer and in turn speak down upon these companies. Similar to the evolved form of competition in school, this overrated form causes pain for all parties and has since diverged from the encouraging nature that the principle of competition was “founded” on.

The AP score for this essay was a 4/6, meaning that it captured the main purpose of the essay but there were still substantial parts missing. In this essay, the writer did a good job organizing the sections and making sure that their writing was in order according to the thesis statement. The essay first discusses how competition is harmful in elementary school and then discusses this topic in the context of business. This follows the chronological order of somebody’s life and flows nicely.

The arguments in this essay are problematic, as they do not provide enough examples of how exactly competition is overrated. The essay discusses the context in which competition is overrated but does not go far enough in explaining how this connects to the prompt.

In the first example, school stress is used to explain how competition manifests. This is a good starting point, but it does not talk about why competition is overrated; it simply mentions that competition can be unhealthy. The last sentence of that paragraph is the main point of the argument and should be expanded to discuss how the anxiety of school is overrated later on in life. 

In the second example, the writer discusses how competition can lead to harmful business practices, but again, this doesn’t reflect the reason this would be overrated. Is competition really overrated because Apple and McDonald’s force you to buy new products? This example could’ve been taken one step farther. Instead of explaining why business structures—such as monopolies—harm competition, the author should discuss how those particular structures are overrated.

Additionally, the examples the writer used lack detail. A stronger essay would’ve provided more in-depth examples. This essay seemed to mention examples only in passing without using them to defend the argument.

It should also be noted that the structure of the essay is incomplete. The introduction only has a thesis statement and no additional context. Also, there is no conclusion paragraph that sums up the essay. These missing components result in a 4/6.

Now let’s go through the prompt for a sample essay from the May 2022 exam . The prompt is as follows:

Colin Powell, a four-star general and former United States Secretary of State, wrote in his 1995 autobiography: “[W]e do not have the luxury of collecting information indefinitely. At some point, before we can have every possible fact in hand, we have to decide. The key is not to make quick decisions, but to make timely decisions.”

Write an essay that argues your position on the extent to which Powell’s claim about making decisions is valid. 

In your response you should do the following:

  • Respond to the prompt with a thesis that presents a defensible position. 
  • Provide evidence to support your line of reasoning. 
  • Explain how the evidence supports your line of reasoning. 
  • Use appropriate grammar and punctuation in communicating your argument.

Sample Student Essay #2:

Colin Powell, who was a four star general and a former United States Secretary of State. He wrote an autobiography and had made a claim about making decisions. In my personal opinion, Powell’s claim is true to full extent and shows an extremely valuable piece of advice that we do not consider when we make decisions.

Powell stated, “before we can have every possible fact in hand we have to decide…. but to make it a timely decision” (1995). With this statement Powell is telling the audience of his autobiography that it does not necessarily matter how many facts you have, and how many things you know. Being able to have access to everything possible takes a great amount of time and we don’t always have all of the time in the world. A decision has to be made with what you know, waiting for something else to come in while trying to make a decision whether that other fact is good or bad you already have a good amount of things that you know. Everyone’s time is valuable, including yours. At the end of the day the decision will have to be made and that is why it should be made in a “timely” manner.

This response was graded for a score of 2/6. Let’s break down the score to smaller points that signify where the student fell short.

The thesis in this essay is clearly outlined at the end of the first paragraph. The student states their agreement with Powell’s claim and frames the rest of their essay around this stance. The success in scoring here lies in the clear communication of the thesis and the direction the argument will take. It’s important to make the thesis statement concise, specific, and arguable, which the student has successfully done.

While the student did attempt to provide evidence to support their thesis, it’s clear that their explanation lacks specific detail and substance. They referenced Powell’s statement, but did not delve into how this statement has proven true in specific instances, and did not provide examples that could bring the argument to life.

Commentary is an essential part of this section’s score. It means explaining the significance of the evidence and connecting it back to the thesis. Unfortunately, the student’s commentary here is too vague and does not effectively elaborate on how the evidence supports their argument.

To improve, the student could use more concrete examples to demonstrate their point and discuss how each piece of evidence supports their thesis. For instance, they could discuss specific moments in Powell’s career where making a timely decision was more valuable than waiting for all possible facts. This would help illustrate the argument in a more engaging, understandable way.

A high score in the “sophistication” category of the grading rubric is given for demonstrating a complex understanding of the rhetorical situation (purpose, audience, context, etc.), making effective rhetorical choices, or establishing a line of reasoning. Here, the student’s response lacks complexity and sophistication. They’ve simply agreed with Powell’s claim and made a few general statements without providing a deeper analysis or effectively considering the rhetorical situation.

To increase sophistication, the student could explore possible counterarguments or complexities within Powell’s claim. They could discuss potential drawbacks of making decisions without all possible facts, or examine situations where timely decisions might not yield the best results. By acknowledging and refuting these potential counterarguments, they could add more depth to their analysis and showcase their understanding of the complexities involved in decision-making.

The student could also analyze why Powell, given his background and experiences, might have come to such a conclusion, thus providing more context and showing an understanding of the rhetorical situation.

Remember, sophistication in argumentation isn’t about using fancy words or complicated sentences. It’s about showing that you understand the complexity of the issue at hand and that you’re able to make thoughtful, nuanced arguments. Sophistication shows that you can think critically about the topic and make connections that aren’t immediately obvious.

Now that you’ve looked at an example essay and some tips for the argumentative essay, you know how to better prepare for the AP English Language and Composition Exam.

While your AP scores don’t usually impact your admissions chances , colleges do care a lot about your course rigor. So, taking as many APs as you can will certainly boost your chances! AP scores can be a way for high-performing students to set themselves apart, particularly when applying to prestigious universities. Through the process of self-reporting scores , you can show your hard work and intelligence to admissions counselors.

That said, the main benefit of scoring high on AP exams comes once you land at your dream school, as high scores can allow you to “test out” of entry-level requirements, often called GE requirements or distribution requirements. This will save you time and money.

To understand how your course rigor stacks up, check out CollegeVine’s free chancing engine . This resource takes your course rigor, test scores, extracurriculars, and more, to determine your chances of getting into over 1600 colleges across the country!

Related CollegeVine Blog Posts

ap english research paper example

AP English Language and Composition: Sample Rhetorical Analysis and Synthesis Questions

April 9, 2024.

AP English Language Sample Rhetorical Analysis and Synthesis Questions

The Rhetorical Analysis and Synthesis Essays are two of the three essays you’ll need to write as part of the AP English Language and Composition Exam . Read on for a sample of each, as well as tips for how to answer them. 

AP English Language and Composition: Sample Rhetorical Analysis Question

Read the following passage published back in 1967 by The New York Times. Then write an essay in which you analyze how the structure of the passage and the use of language help convey the writer’s views.

Sample Question Instructions:

  • Respond to the prompt with a thesis that may establish a line of reasoning.
  • Select and use evidence to develop and support the line of reasoning.
  • Explain the relationship between the evidence and the thesis.
  • Demonstrate an understanding of the rhetorical situation.
  • Use appropriate grammar and punctuation in communicating the argument.

Americans and Western Europeans, in their sensitivity to lingering problems around them, tend to make science and progress their scapegoats. There is a belief that progress has precipitated widespread unhappiness, anxieties, and other social and emotional problems. Science is viewed as a cold mechanical discipline having nothing to do with human warmth and the human spirit. 

But to many of us from the nonscientific East, science does not have such repugnant associations. We are not afraid of it, nor are we disappointed by it. We know all too painfully that our social and emotional problems festered long before the age of technology. To us, science is warm and reassuring. It promises hope. It is helping us at long last gain some control over our persecutory environments, alleviating age-old problems—not only physical but also, and especially, problems of the spirit.

Shiraz, for example, a city in southern Iran, has long been renowned for its rose gardens and nightingales; its poets, Sadi and Hafiz; and its mystical, ascetic philosophy, Sufism. Much poetry has been written in glorification of the spiritual attributes of this oasis city. And to be sure, Shiraz is a green, picturesque town, with a quaint bazaar and refreshing gardens. But in this “romantic” city thousands of emotionally disturbed and mentally retarded men, women, and children were, until recently, kept in chains in stifling prison cells and lunatic asylums. 

Every now and again, some were dragged, screaming and pleading, to a courtyard and flogged for not behaving “normally.” But for the most part, they were made to sit against damp walls, their hands and feet locked in chains, and thus immobilized, without even a modicum of affection from their helpless families and friends, they sat for weeks and months and years—often all their lives. Pictures of these wretched men, women, and children can still be seen in this “city of poetry,” this “city with a spiritual way of life.” 

It was only recently that a wealthy young Shirazi who, against the admonitions of his family, had studied psychology at the University of Tehran and foreign universities, returned to Shiraz and after considerable struggle with city officials succeeded in opening a psychiatric clinic, the first in those regions. After still more struggle, he arranged to have the emotionally disturbed and the mentally retarded transferred from prison to their homes, to hospitals, and to his clinic, where he and his staff now attend them. 

They are fortunate. All over Asia and other backward areas, emotionally disturbed men and women are still incarcerated in these medieval dungeons called lunatic asylums. The cruel rejection and punishment are intended to teach them a lesson or help exorcise evil spirits. 

The West, still bogged down in its ridiculous romanticism, would like to believe that emotional disturbances, dope addiction, delinquency are all modern problems brought on by technological progress, and that backward societies are too spiritual and beautiful to need the ministrations of science. But while the West can perhaps afford to think this way, the people of backward lands cannot. . . . 

. . .The obstacles are awesome, the inertia too entrenched, the people’s suffering too anguished, their impatience too eruptive. Moreover, the total cultural reorganizations such as Asia and Africa are undergoing inevitably engender their own temporary dislocations and confusions. But their goals, the direction, remain constant. We are on the move, however awkwardly at first, to a saner, better world.

How to Answer the AP English Language and Composition Rhetorical Analysis Question

Go back to the original question, which asks you to analyze two features of the passage: (1) its structure, or organization, and (2) its language. The first aspect is fairly specific. As you read the passage, you need to observe what the author discusses first, second, third, and so on. Your essay should explain not only the order of ideas but the reasons the author may have chosen that order. 

The second part of the question is more general. It invites you to analyze the use of language, which may include the author’s choice of words (diction), syntax (word order), figures of speech, use of evidence (such as statistics or logical reasoning), sentence structure, rhythm, sound, tone, or just about any other characteristics of style and rhetoric you choose. 

Although the question directs you to write about two different aspects of the passage, the essay itself should be unified. That is, a good essay should not consist of, say, two disparate paragraphs, one exclusively devoted to structure and another to language. Rather, the essay should include material that shows the interrelationship of structure and language in the passage and how those elements contribute to the meaning and effect of the passage. This might be covered in a separate paragraph, or it could be woven into the overall fabric of the essay. 

Before you begin to write, read the passage at least twice: once for an overview and once as you write your analysis. You may notice early on that the opening paragraph contains generalizations about Westerners’ concepts of science and progress. Then the author contrasts the Western view of science and progress with the Eastern view. Immediately, you see that the author, by using the first-person pronoun (as in “many of us”) is speaking from the perspective of an Easterner. Consequently, his discussion of Eastern views is apt to come across as more well-informed, more authoritative, perhaps more personal. 

To support his position, the author gives an extended example—the city of Shiraz—to illustrate just how different the East is from the West. The description and vivid images of Shiraz memorably convey the idea that the “spiritual way of life” has a side to it that many Westerners don’t know about. This is the heart of the passage. The use of quotation marks around “romantic” and “city of poetry” is meant to point out the discrepancy between the idealized and real versions of Shiraz. 

Nearing the end, the author reiterates his initial contrast between West and East, with emphasis on the East. The last paragraph offers a generalized statement about conditions in Asia and Africa, reminding the reader of the contrast made at the very beginning of the passage. Tying the end to the beginning of the passage creates a sense of unity—a desirable feature in any piece of writing.

AP English Language and Composition: Sample Argument Question

The following paragraph is adapted from Mirror for Man, a book written by anthropologist Clyde Kluckhorn in the middle of the twentieth century. Read the passage carefully. Then, write an essay that examines the extent to which the author’s characterization of the United States holds true today. Use appropriate evidence to support your argument. 

Sample Question Instructions: 

  • Respond to the prompt with a thesis that may establish a line of reasoning. 
  • Select and use evidence to develop and support the line of reasoning. 
  • Explain the relationship between the evidence and the thesis. 
  • Demonstrate an understanding of the rhetorical situation. 

Technology is valued as the very basis of the capitalistic system. Possession of gadgets is esteemed as a mark of success to the extent that persons are judged not by the integrity of their characters or by the originality of their minds but by what they seem to be—so far as can be measured by their wealth or by the variety and material goods which they display. “Success” is measured by their investments, homes, and lifestyles— not by their number of mistresses as in some cultures.

How to Answer the AP English Language and Composition Argument Question

Whether you agree, disagree, or have mixed views on the content of the passage, your job is to write a convincing argument that expresses your opinion. Initially, the word argument may suggest conflict or confrontation. But rest assured that your essay need not be combative. Rather, make it a calmly-reasoned explanation of your opinion on a debatable subject. Your goal is to persuade the reader that your opinion, supported by examples, facts, and other appropriate evidence, is correct. 

If you have strong feelings about the topic, of course you should state them in your essay. But express them in calm, rational language. Be mindful that the essay should not be an emotional rant for or against the issue. 

Consider first whether you agree with Kluckhorn’s definition of “success.” Is it, as Kluckhorn asserts, measured by income and material possessions? Or do you think that a more accurate standard of success in today’s America should be determined by less tangible criteria—things such as happiness or self-respect? Or do you stand somewhere in between those two extremes? 

The actual position you take on the issue is less crucial than your ability to support it fully by drawing from your knowledge, background, experience, or observation. Regardless of your position, be sure to include more than one example. An argument that relies on a single example, however compelling, will fall flat. 

In the prompt, Kluckhorn’s notion of success seems to refer broadly to American society. Resist responding in kind. That is, a short essay shouldn’t focus on the whole of society but only on an identifiable segment—perhaps college-educated professionals or urban, blue- collar Americans. The point is that a narrowly focused essay on a limited topic will always turn out better than one that tries to cover too much ground in just a few paragraphs.

AP Biology Resources

  • About the AP Biology Exam
  • Top AP Biology Exam Strategies
  • Top 5 Study Topics and Tips for the AP Biology Exam
  • AP Biology Short Free-Response Questions
  • AP Biology Long Free-Response Questions

AP Psychology Resources

  • What’s Tested on the AP Psychology Exam?
  • Top 5 Study Tips for the AP Psychology Exam
  • AP Psychology Key Terms
  • Top AP Psychology Exam Multiple-Choice Question Tips
  • Top AP Psychology Exam Free Response Questions Tips
  • AP Psychology Sample Free Response Question

AP English Language and Composition Resources

  • What’s Tested on the AP English Language and Composition Exam?
  • Top 5 Tips for the AP English Language and Composition Exam
  • Top Reading Techniques for the AP English Language and Composition Exam
  • How to Answer the AP English Language and Composition Essay Questions 
  • AP English Language and Composition Exam Sample Essay Questions
  • AP English Language and Composition Exam Multiple-Choice Questions

AP Human Geography Resources

  • What’s Tested On the AP Human Geography Exam?
  • AP Human Geography FAQs
  • AP Human Geography Question Types and Strategies
  • Top 5 Study Tips for the AP Human Geography Exam

FOLLOW ALONG ON SOCIAL

  • How It Works
  • PhD thesis writing
  • Master thesis writing
  • Bachelor thesis writing
  • Dissertation writing service
  • Dissertation abstract writing
  • Thesis proposal writing
  • Thesis editing service
  • Thesis proofreading service
  • Thesis formatting service
  • Coursework writing service
  • Research paper writing service
  • Architecture thesis writing
  • Computer science thesis writing
  • Engineering thesis writing
  • History thesis writing
  • MBA thesis writing
  • Nursing dissertation writing
  • Psychology dissertation writing
  • Sociology thesis writing
  • Statistics dissertation writing
  • Buy dissertation online
  • Write my dissertation
  • Cheap thesis
  • Cheap dissertation
  • Custom dissertation
  • Dissertation help
  • Pay for thesis
  • Pay for dissertation
  • Senior thesis
  • Write my thesis

170 AP Research Topics For Your Paper

170 AP Research Topics

As you may already know, AP Seminar is across multiple disciplines of study. It primarily explores how students can achieve critical thinking abilities through collaboration and thorough academic research. AP Seminar also helps the student understand real-world issues from different lenses.

This way, you get to understand how the world works through multiple perspectives. AP Seminar usually covers team projects, individual papers, oral presentations, and the exam at the end of the course. How do you prepare for your project, paper, or oral presentation without essential AP research paper topics?

It would help if you have AP research topics ideas and perhaps, AP research questions. You need AP seminar topics that help you develop your analytical and creative research and writing skills. You will find them in this content. However, before that, what must be found in a good AP research paper?

Characteristics of a Good AP Research Paper

Writing a good research paper requires analyzing previous academic papers and improving your knowledge about what you want to write. While writing your research paper, take notes of these:

  • A Good Papers Must be Relevant in the Long Term This means you should not write on something that will no longer be relevant in two years. In other words, think about Shakespeare. Or better still, think about writing something that many other writers and researchers will reference. Your logic must have no noticeable unclear gap. Your research must also pass through a thorough assessment process to ascertain that everything in your paper is relevant to the present and the future. If you are writing something about history, you should apply logic and thought to the sources you pick and write about them.
  • Backup Your Research with Valid Sources Every research paper without an external source is poor. Especially when you quote facts, your good sources make your paper appealing and worthy of application or study by different generations. Your paper must include all the facts it needs to pass each point. This is also important because having solid references means that you consulted excellent sources and adopted their information while writing your paper.
  • Have Great Research Questions Research questions sometimes turn your paper around. This is because your AP research questions must be well thought of and dynamic to the issues you’re writing about. When you choose from AP seminar themes available to you or the AP research topic ideas of your choice, asking yourself questions your research will answer makes it easy to focus. Your research questions influence your research, and they make writing easy too.
  • Your Research Must be Specific You can only master how to stay relevant by writing on specific issues. For example, if you want to write about coronavirus, you should not make the mistake of treating a general topic. Instead, narrow it down to your environment or a controversial issue. The Atlantic recently published a long-form essay about a hospital overwhelmed with patients. They eventually had to use their ambulances for some operations. That is an example of a research that treats a specific issue, not a generic one.
  • Your Research Must be Well Structured and Concise This is perhaps the most critical part of your work. If you have an excellent paper, it will only reflect when you have a great structure. As you know, your paper must have an introduction, the main body, and the conclusion.
The introduction is where you discuss the general concepts and the target of your paper. You can also discuss why it is essential to examine the topic; this could be your problem statement. Your main body is where you disassemble and reassemble your facts, make your arguments, and pass your message. Note that you will answer all your research questions in this section, which is why the section is most important while writing a paper. It will help if you back every fact up with solid references. Your conclusion is where you sum everything you’ve raised and discussed. You may attach a quick call to action to the necessary authorities, depending on what you’ve written. However, if you have just sensitized your readers with your project, your conclusion should end with a perfect sentence or paragraphs that stays with your readers.

Now that you know all these, consider these AP seminar research paper topics for your project, paper, or oral presentation. On the other hand, remember you can buy thesis online from our professional helpers team and never worry about your grades.

AP Seminar Themes

AP Seminar themes are the usual subject of discourse that often appear in any AP Seminar. These themes unite all the courses you study during your one year long course. You can consider these AP research topics:

  • The ethics of algorithms on social media
  • Controversial supreme court ruling based on public opinion
  • Environmental injustice and consequences on real estate
  • The challenges of typo errors
  • Overview of genetics
  • The cause of food waste
  • Universal adult suffrage and the fault in the system
  • #MeToo movement and the challenges
  • The issues on wealth inequality
  • Agree or disagree with Francis Fukuyama’s The End of History.”
  • Social class exists in communist societies
  • Communism is the feasible system of the future
  • Art and education
  • Music and education
  • The significance of entertainment in the workplace.

AP Seminar Topics

These are direct AP seminar research topics that you can use for your paper. The best topics are often intriguing enough to impress your teachers for your dissertation or thesis. You can use these to create the perfect paper:

  • Food waste management in Ukraine
  • How the naval power of China is not enough over Taiwan and its allies
  • Low lying islands and global warming
  • The differences in Chinese GDP between 1978 and 2020
  • How global warming affects countries that contribute less to it
  • The rise in the creation of energy-efficient cars
  • Genetically modified plants ruin nature: discuss
  • Pesticides cause more harm than good: discuss
  • Saving money in college is underrated
  • Significance of electric cars to the future
  • A study on three robots and the prospects for the future workplace
  • Gun control is impossible: discuss
  • Intensive farming may not end hunger: discuss
  • Examine the output of three countries and their wealth
  • How much has the world achieved gender equality?
  • Discuss why the demand for tech is a challenging issue
  • Compare and contrast the labor market in the US and Germany
  • Would you say China is the superpower of sustainable batteries
  • Study the rising sectors in the world and their consequences
  • Will petroleum, not batteries, be vital in 2030?
  • Homework is overrated
  • What are the controversies around the statement “I am self-taught.”
  • Significance of noise pollution
  • Quantum entanglement: discuss
  • China and its tech products
  • UK and software growth
  • Is Africa forgotten?
  • Challenges between black Americans and African Americans
  • Aside from COVID-19 and omicron, what are globally threatening issues?
  • Abortion is unethical: discuss.

AP Research Topic Ideas

As a research student, you may also want to consider good ideas to develop for your paper. Your AP research topic ideas must aid your analytical skills. It should also help you create interesting perspectives about issues. You can consider:

  • How the ozone layer protects the planet
  • The best way to address global warming
  • A study of Elon Musk’s education system
  • A study of the Swiss educational system
  • Challenges of education in the UK
  • Challenges on education in the US
  • Study how prison changes people
  • Discuss recent aquatic discoveries
  • How to answer to Brazil’s deforestation
  • The effects of deforestation on global health
  • The rise and evolution of taxes
  • Is the US the world police?
  • Trace world religions: is there a peaceful religion?
  • Discuss how Jews weaponized the Holocaust narrative in Palestine
  • Discuss why some courses are compulsory
  • Examine the problem with obesity
  • Discuss what it means to be in denial
  • Analyze US’s withdrawal from Afghanistan
  • Human right over national security: discuss
  • Is the EU sustainable?
  • Discuss the activities of CNN’s Jeff Zucker
  • Why is history important?
  • Is philosophy a foundational discipline for all disciplines?
  • Examine measures to prevent a WWIII
  • Lessons from the failure of Right 2 Respond in Syria
  • Discuss the challenges of mandatory service
  • Was the Cold War indeed a cold war?
  • What does the world know about Antarctica?
  • Debt diplomacy: discuss
  • An overview of terrorism.

AP World History Research Paper Topics

World history can be broad yet interesting. World history is interesting when you apply rational thought and philosophy into the aspects of the past that matter and those that don’t. These are some good AP world history research paper topics:

  • Effects of the industrial revolution
  • The evolution of German industries
  • The Crusaders are the same as the Jihads: discuss
  • The struggle for democracy
  • French revolution and consequences on Europe
  • The US independence and consequences on Europe
  • The decolonization of Europe led to the colonization of Africa
  • British imperialism and challenges
  • Mongols and relevance in history
  • What are the similarities and differences in the civilizations of Ancient Egypt and Mesopotamia
  • Discuss the hegemony of the Ottoman Empire
  • Discuss the evolution of contemporary politics
  • Examine history and religion
  • How Hitler could have won WWII
  • How Asia changed post-WWI
  • The symbols in Ancient Egypt
  • The symbols in Mesopotamia civilization
  • How agricultural revolution affect the world
  • The rise of industries in Japan
  • The growth of armament and present challenges
  • Water challenges in the Middle East
  • The Middle East is a vital region to the globe: discuss
  • Would you say Muammar Gaddafi of Libya was a great leader?
  • Examine the disunity in the Arab League
  • Mortification in Ancient Egypt
  • The Jews and the world
  • The cyclical nature of history
  • Sexual revolution and trends
  • Nudity and its politics
  • Child trafficking in India.

AP US History Research Paper Topics

To engage in more direct research, you may want to integrate your thoughts into interesting AP research topics. These are topics about current and past events in US history:

  • Differences between presidents Trump and Clinton
  • The US and the Great Depression
  • US interference in the Suez Canal crisis
  • US foreign policy to Libya under Gaddafi
  • The US before and after 9/11
  • Religions of the Native Americans
  • The fiction in Salem Witch Trials
  • Women’s role in America’s independence
  • Women’s role in the Renaissance Europe
  • Sexual revolution as feminism
  • Industrial revolution in America
  • The causes of slavery in the Americas
  • America will always fight wars: discuss
  • The reasons why America built up its armies
  • The evolution of labor unions in America
  • An overview of the Cuban Missile Crisis
  • The US and social media policy
  • US and gender inequality
  • Corruption in the US
  • Gangsterism in the US
  • George Floyd and the consequences of his death
  • The effects of the Mississippi River Flood
  • CIA and the murder of Congolese Patrice Lumumba: justifiable?
  • Ideological challenges in US history
  • Indigenous technology in the American Civil War
  • The effect of Hurricane Katrina
  • An overview of a mass shooting in any location of your choice
  • Environmental crisis in the US
  • Was the Cold War necessary?
  • Criticize the bombing of Hiroshima and Nagasaki.

AP Capstone Research Topics

AP Capstone involves the assessment and overview of all the courses during your AP Seminar. It examines your knowledge of different theories and how you can apply them. You can choose these topics for your paper:

  • The US and its ethnicities
  • US and religious duplicity
  • Vaccines and their benefits in the US
  • How does the US control the UN
  • Bullying and its impacts
  • How social media affects students
  • Food insecurity and challenges of the contemporary world
  • The future of the automobile
  • The future of automation
  • Does Elon Musk do something similar to Mark Zuckerberg?
  • Extreme sports: what’s the thrill?
  • The history of vaccines
  • Is America truly democratic
  • Discuss the overaction of lawmakers on the Janet Jackson Superbowl performance saga
  • Is the UN a toothless bulldog?

AP Biology Research Topics

Biology is an integral part of the world and a relevant course in your AP Seminar. You can choose to discuss any of the following interesting topics:

  • Critically analyze how the human body works.
  • Discuss the correlation between the way Ancient Egyptians preserved their dead and the way it is done today
  • How has the research and writings of the Romantic Era achieved some changes in the biology of contemporary society?
  • What is the most common thing that weakens the human immune system?
  • The HIV/AIDS epidemic and the vulnerability of the human society
  • The scientists’ struggle in preventing the spread of the virus before, during, and after COVID-19
  • Assess the importance of genetics, if any, in the creation of robots with emotional abilities like humans
  • Assess how stress affects the immune system and how a strenuous activity as sex helps ease stress
  • Speak with any five persons and evaluate why some people have refused to take the COVID-19 vaccine
  • Discuss how bacteria affect the body and its consequences
  • Evaluate how animals communicate
  • Trace the evolution and trends in the discovery that the heart controls human life, not the brain
  • A study of how white blood cells work and fight diseases
  • Should DNA tests be made accessible for the public?
  • Cell division and how to teach how it works to pupils
  • Examine the process of photosynthesis on plants
  • The trends and discoveries of cancer treatments
  • Read five pieces of literature on different perspectives on genetics and appraise them.
  • Analyze how the red blood cells convey oxygen
  • Appraise the structure of the human body.

Don’t Want To Write Your Paper Yourself?

Now that you have the best AP research topics, you can create an interesting paper. However, if you need help with dissertation or college thesis, we offer thesis writing service that will wow your professors.

We are a team of professional writers and researchers with years of academic writing experience. We are based online for accessibility sake. We have teachers and professors amongst us, most of whom will create your custom paper.

You have the luxury of choosing any of our expert writers to work on your paper or presentation. We have consistently created quality papers for hundreds of students over the years. Our professional writers write fast to meet deadlines at a cheap rate. In other words, we are pocket-friendly.

pros and cons topics

Leave a Reply Cancel reply

Your email address will not be published. Required fields are marked *

Comment * Error message

Name * Error message

Email * Error message

Save my name, email, and website in this browser for the next time I comment.

As Putin continues killing civilians, bombing kindergartens, and threatening WWIII, Ukraine fights for the world's peaceful future.

Ukraine Live Updates

  • Study Notes
  • College Essays

AP English Notes

  • Sample Essays
  • Rhetorical Terms
  • Bonus Knowledge
  • Sample Research Paper - "The Patriot Act"

The Patriot Act Threatens Fundamental American Freedoms

by Feross Aboukhadijeh - 11th grade

Forty-five days after the September 11 terrorist attacks on the United States, Congress passed the USA PATRIOT Act, also known as the “Uniting and Strengthening America by Providing Appropriate Tools Required to Intercept and Obstruct Terrorism” Act, or more simply, the Patriot Act. The Patriot Act was created with the noble intention of finding and prosecuting international terrorists operating on American soil; however, the unfortunate consequences of the Act have been drastic. Many of the Patriot Act’s provisions are in clear violation of the U.S. Constitution—a document drafted by wise men like Benjamin Franklin, James Madison, Alexander Hamilton, and George Washington in order to protect American rights and freedoms. The Patriot Act encroaches on sacred First Amendment rights, which protect free speech and expression, and Fourth Amendment rights, which protect citizens against “unwarranted search and seizure” (Justice). The Patriot Act authorizes unethical and unconstitutional surveillance of American citizens with a negligible improvement in national security. Free speech, free thinking, and a free American lifestyle can not survive in the climate of distrust and constant fear created by the Patriot Act.

The great American patriot Robert F. Kennedy once said in his famous “Day of Affirmation Address” that the first and most critical element of “individual liberty is the freedom of speech; the right to express and communicate ideas, to set oneself apart from the dumb beasts of field and forest . . .” Modern American politicians and lawmakers, it seems, have lost sight of the important ideals that Kennedy spoke about and upon which this country was founded—ideals like civil rights, personal freedom, and the right to privacy. No longer can a newspaper editor publish an article that is critical of the government—even if it is legal—without fear that Big Brother may begin to survey his every thought and action. This may very well be the most frightening aspect of the Patriot Act: the fact that the Act allows the government to spy on any of its citizens, not just the bad ones. The Patriot Act does not demand sufficient proof that alleged “suspects” are engaged in criminal activity before authorizing government surveillance. Even upstanding American citizens can become targets of Federal Bureau of Investigation (FBI) surveillance simply because of the manner in which they exercise their First Amendment rights (Beeson). Simply put, the Patriot Act fails to secure American liberties; in reality, the Act exposes Americans to potential abuses of power by creating an environment that encourages government corruption, secrecy, fraud and discrimination while using “national security” as a pretense for violating basic Constitutional rights like privacy and free speech. As the century drags on, it is becoming painfully obvious that the Patriot Act has actually moved the United States further away from an ideal democratic society since its passage in October of 2001.

Ever since 1776, when American colonists first abandoned their ties with Britain to create an independent nation, American citizens have always cherished basic rights like freedom of speech, freedom of the press, and protection from unreasonable searches and seizures (United States). But after the unpredictable events of September 11, 2001, many citizens began to feel that they should give up some of their cherished rights in order to punish the perpetrators of the attacks and avoid future tragedies. An overwhelming sense of national unity overtook the country and Americans united to face the newly discovered threat of terrorism in a modern age. The President’s approval rating increased from 54% to 86%—its highest level ever—in a matter of days (Ruggles). The American people rallied behind the Federal government and provided support. Tragically, Congress drafted the Patriot Act and decreed that it would be the solution to America’s problems. According to Congress, the Patriot Act would protect America from its enemies who operated on American soil. Many Americans unquestionably accepted the Act to avoid the risk of being labeled “unpatriotic.” However, thousands of far-seeing Americans publicly questioned the actions of the government, but their cries were not heard. When the House of Representatives sent the Patriot Act to the Senate, it passed with a vote of 96-1. Peter Justice put it best when he said that “. . . the climate of fear in the weeks after the September 11 attacks and the haste with which the Patriot Act was passed allowed some of its more controversial aspects to escape adequate congressional scrutiny.” Clearly, the “fear frenzy” that took place after the September 11 attacks caused Americans to sacrifice essential civil rights in exchange for a sense of security.

The only Senator to vote against the Patriot Act was Senator Feingold. Feingold is significant because he was the only Senator to fight against the Patriot Act before it was signed into law. The arguments that he made against the Act during September and October of 2001 continue to point out the negative effects the Act has had on American life and will continue to have moving forward in the twenty-first century. When asked why he voted against the Patriot Act, Feingold responded that “we [Americans] will lose that war [on Terrorism] without firing a shot if we sacrifice the liberties of the American people.” Essentially, Feingold argued that the Patriot Act is counter-productive: if government “security” is meant to protect American liberties, then the American people should not have to sacrifice their liberties to purchase security. What purpose will “security” serve if there are no liberties left to defend? If the Federal government curtails American liberty, then security is rendered worthless. Colonial statesman Benjamin Franklin once said that “those who would give up essential liberty to purchase a little temporary safety deserve neither liberty nor safety.” According to Franklin, real American patriots constantly question their government’s intentions in order to ensure that their elected politicians are keeping the “core of American values and principles” at heart while in office (Justice). The Patriot Act does not keep the interests of American citizens in mind because it sacrifices crucial civil rights that have been guaranteed by the Bill of Rights ever since 1776 (United States).

There is no question that the Patriot Act is unconstitutional. The Act violates the fundamental American ideal of “checks and balances” on government power. Normally, the government can not conduct a search of a citizen’s residence without obtaining a warrant and demonstrating a reason to believe that the suspect has committed (or may commit) a crime. But the Patriot Act violates the Fourth Amendment by allowing the government to conduct searches without a warrant—for just about any reason. If the FBI is ever questioned about such activity, shrewd FBI officials simply state that the investigation is crucial to national security, and they are permitted to continue with the operation. In more recent years the situation has improved somewhat, however. Now, before conducting a search, the FBI must obtain a warrant from a secret Foreign Intelligence Surveillance Court (FISA). Ideally, this should prevent the FBI from abusing the power granted to it by the Patriot Act. However, in its twenty-two years of existence the FISA court has only rejected six search warrants out of the 18,747 requested since the court’s creation (“Newstrack”). This means that if the FBI decides it wants to spy on a certain American citizen, it will most likely be able to do so, even without sufficient evidence.

Certainly, the United States government needs to have the power to monitor suspected terrorists—no upstanding American citizen is arguing about that—but the problem lies in the manner in which government monitoring occurs. The Patriot Act fails to strike a desirable balance between protecting American lives against the threat of terrorism and protecting the rights of Americans against potential government abuse (“Reform”). Particularly upsetting about the Act are several critical provisions designed to widely expand government power with limited “checks and balances” and nearly limitless potential for abuse.

Section 213 is one such provision which greatly expands the power of the Federal government. Section 213 of the Patriot Act authorizes law enforcement agents to conduct “sneak-and-peek” operations in a U.S. resident’s home. This provision violates the Fourth Amendment by failing to require that those persons who are the subject of search orders “be told that their privacy has been compromised” (“Reform”). If an individual does not know that the government has been in his home then he will be unable to verify that the government conducted a reasonable search using a valid warrant. If the government indeed did overstep its bounds, the individual will have no means to take recourse against the government. After all, how can a person protect their rights if they do not know that their rights have been violated? Section 213 erodes the “sacred rights of western society” as described by Kennedy, and reduces U.S. civil rights to nearly the same level as those of the Nazi Socialists in Russia during the 1930s and 40s.

Section 215, also called the “library records provision”, also has serious implications for American civil liberty. Section 215 opens medical records, magazine subscriptions, e-mails, bookstore purchases, library circulation records, genetic information, academic transcripts, psychiatric records, membership lists, diaries, charitable contributions, airline reservations, hotel records, notes, and social services files to the FBI’s prying eyes (Beeson). For example, the FBI can request the names of all the patrons that have checked out a certain book from the library, simply because they do not like the topic of that particular book. Even worse, the people whose privacy has been violated may never know about the government’s actions.

Section 505 is another particularly threatening provision of the Patriot Act. Section 505 facilitates the use of “national security letters”, or NSLs, in federal investigations. NSLs are a form of administrative subpoena that legally compel an entity or organization to turn over personal records and information about certain individuals. Previously, the FBI could only use NSLs to access records of foreign agents and known terrorists, but Section 505 of the Patriot Act adds non-terrorism suspects to the list of entities that the FBI can use NSLs to spy on (“Controversial”). The problem with this is that NSLs are substantially easier to obtain than regular subpoenas; NSLs do not have to be authorized by a judge like normal subpoenas—they merely must be signed by certain key FBI agents. This means that the FBI can use NSLs to illegally obtain information about an American citizen who may be involved in some sort of crime. While many Patriot Act supporters may argue that Section 505 is beneficial because the FBI can more easily obtain information about all types of criminals, the truth is that Section 505 violates the Fifth Amendment’s “due process of law” clause. According to the Fifth Amendment, no person should be “. . . deprived of life, liberty, or property, without due process of law . . .” Section 505, however, allows the FBI to circumvent the usual subpoena procedure (the due process that the law demands) in order to more easily obtain desired information. This means that NSLs can legally be used to obtain information about ordinary criminals like robbers, shoplifters, and drug users—and even people who have presented little or no evidence of wrongdoing. NSLs are extremely serious legal weapons and should be reserved for only the most serious of crimes, like terrorism.

While the potential for government abuse of the Patriot Act is all too clear, another alarming fact is that the Patriot Act fails to secure American liberties—proving that the Act has failed in its purpose. According to Donna Lieberman, Executive Director of the NYCLU, “Effective law enforcement in the aftermath of September 11 does not call for a return to the bad old days when there was open season on dissent and dissenters . . . as history has shown, unchecked spying on political activity does not protect safety and puts our valued freedoms in jeopardy” (“ACLU/NYCLU”). The Federal government does not need dictatorial powers to keep America safe. Kennedy would have opposed the Patriot Act for the same reasons that he opposed communism. Kennedy said “I am unalterably opposed to communism because it exalts the state over the individual . . . and because its system contains a lack of freedom of speech, of protest, of religion, and of the press, which is characteristic of a totalitarian regime . . .” It is frighteningly un-American to assume that giving politicians authoritarian powers will make America safer. Furthermore, Kennedy would have argued that the way to oppose terrorists is to “enlarge individual human freedom”—not take it away. By allowing the Federal government to take away freedoms and civil rights, Americans are actually helping the terrorists to erode the ideals of the American system. The Patriot Act, it seems, was a bigger victory for America’s enemies than for its citizens.

Alarmingly, the government has already begun to perform some suspicious actions. The FBI is keeping secret “even the most basic information” about FBI surveillance (“Reform”). For example, the FBI classified information that should have been available to the public—information that would have shown how often the FBI has spied on people based on the manner that they exercise their First Amendment rights. Although this action in and of itself does not prove that the FBI or the Federal government has explicitly broken the law, it does hint that the government is trying to hide its activities from public scrutiny. The American Civil Liberties Union (ACLU) wrote that “. . . the few known cases of rights violations under the Patriot Act are likely the tip of the iceberg in terms of abuses of the investigative powers the government has under the Patriot Act because most such investigation is conducted secretly.” In other words, the few verified cases of government abuse of the Patriot Act may indicate many more abuses which are not disclosed to the public.

Although some Americans may say, “I don’t mind the Patriot Act because I have nothing to hide from the government”, this thinking is flawed for several reasons. First of all, if the American people know that their actions and communications are being monitored they will feel less comfortable expressing their thoughts and exercising their rights to free speech and free thinking; this is especially so if the person’s thoughts are not what the government wants them to think. Second, by eroding American civil rights in order to obtain a sense of security, Americans are actually helping the terrorists to achieve their mission of destroying democratic ideals in the western world. The last and most compelling reason to oppose the Patriot Act is the fact that it is a direct attack on American ideals. The Patriot Act essentially destroys the protections offered by the First and Fourth Amendments and exposes Americans to potential abuses at the hands of Big Brother.

Protecting Americans from foreign threats is critical; the Federal government should do whatever it takes to keep its citizens safe, but it should never infringe upon their civil rights. No doubt, the Patriot Act represents an emerging trend in American government today—a trend of sacrificing the American Creed’s ideals in exchange for security. Americans fought the Revolutionary War to earn basic liberties that they felt were their God-given rights—rights that no humans should live without. Americans should not so easily relinquish the rights and liberties cherished for so long as the cornerstone of American society for the mere illusion of security.

Works Cited

"ACLU/NYCLU Mobilize Members and Supporters to Keep America ‘Safe and Free’." NYCLU . ACLU Foundation. 5 Mar. 2007 <http://www.nyclu.org/safe_free101602.html>.

Beeson, Ann, and Jameel Jaffer. "Unpatriotic Acts: the FBI's Power to Rifle Through Your Records and Personal Belongings Without Telling You." ACLU . July 2003. ACLU Foundation. 25 Feb. 2007 <http://www.aclu.org/FilesPDFs/spies_report.pdf>.

"Controversial Provisions of the USA Patriot Act." Facts on File News Services . 14 Apr. 2006. 5 Apr. 2007 <http://www.2facts.com/ICAH/Search/has00001371.asp>.

Feingold, Russell. "Senator Feingold's Speech Explaining Why He Voted Against the Patriot Act (Excerpts)." Facts on File News Services . 12 Oct. 2001. 8 Feb. 2007 <http://www.2facts.com/ICAH/Search/had00000278.asp>.

Justice, Peter. "USA Patriot Act." Facts on File News Services . 14 Apr. 2006. 8 Feb. 2007 <http://www.2facts.com/ICAH/Search/haa00001370.asp>.

Kennedy, Robert F. "Day of Affirmation Address." University of Capetown, Capetown. 6 June 1996. 5 Apr. 2007 <http://www.jfklibrary.org/Historical+Resources/Archives/ Reference+Desk/Speeches/RFK/Day+of+Affirmation+Address+News+Release+Page+2.htm>.

“Newstrack - Top News." United Press International 26 Dec. 2005. 5 Apr. 2007 <http://www.upi.com/NewsTrack/Top_News/2005/12/26/ bush_was_denied_wiretaps_bypassed_them/>.

"Reform the Patriot Act: Section 215." ACLU . ACLU Foundation. 25 Feb. 2007 <http://action.aclu.org/reformthepatriotact/215.html>.

Ruggles, Steven. "Historical Bush Approval Ratings." 5 Mar. 2007. University of Minnesota. 5 Mar. 2007 <http://www.hist.umn.edu/~ruggles/Approval.htm>.

United States. The Bill of Rights . 5 Mar. 2007 <http://usinfo.state.gov/usa/infousa/facts/funddocs/billeng.htm>.

Works Consulted

"A Case of Justice That Stinks: the Administration's Forced Resignations Among U.S. Attorneys Tell the Tale of the Corrupting Influence of Unchecked Power." The Roanoke Times . 21 Jan. 2007. McClatchy-Tribune Business News. 8 Feb. 2007 <http://search.ebscohost.com/login.aspx?direct=true&db=nfh&AN=2W62W63048083835&site=src-live>.

Berlau, John. "Show Us Your Money: the USA PATRIOT Act Lets the Feds Spy on Your Finances. But Does It Help Catch Terrorists?" Reason (2003). 8 Feb. 2007 <http://findarticles.com/p/articles/mi_m1568/is_6_35/ai_109085440/pg_1>.

"Newstrack - Top News." United Press International 26 Dec. 2005. 5 Apr. 2007 <http://www.upi.com/NewsTrack/Top_News/2005/12/26/ bush_was_denied_wiretaps_bypassed_them/>.

"Robert F. Kennedy." Britannica Concise Encyclopedia . Encyclopædia Britannica, Inc. 8 Feb. 2007 <http://www.answers.com/topic/robert-f-kennedy>.

"Robert F. Kennedy." Who2? Biographies . 8 Feb. 2007 <http://www.answers.com/topic/robert-f-kennedy>.

United States. Cong. Uniting and Strengthening America by Providing Appropriate Tools Required to Intercept and Obstruct Terrorism (Usa Patriot Act) Act of 2001 . 107th Cong. 26 Oct. 2001. 8 Feb. 2007 <http://frwebgate.access.gpo.gov/cgi-bin/getdoc.cgi?dbname=107_cong_public_laws&docid=f:publ056.107>.

"USA Patriot Act (2001)." The United States At War . 26 Oct. 2001. ABC-CLIO. 8 Feb. 2007 <http://www.usatwar.abc-clio.com/Research/Display.aspx?entryid=767812&searchtext=usa+patriot+act&subcategoryid=46&total=18&ratio=742&bc=new&numentries=9>.

ap english research paper example

You just finished Sample Research Paper - "The Patriot Act" . Nice work!

Previous Essay Next Essay

Tip: Use ← → keys to navigate!

How to cite this note (MLA)

More ap english sample essays.

  • Sample Definition Essay - "Success"
  • Sample Character Analysis Essay - "Hamlet"
  • Sample Thesis Statements
  • Essay Tips: Style Analysis - Tone of Voice Words
  • Sample Compare and Contrast Essay - "Lincoln/Douglass"
  • Sample Literary Devices Essay - "Things Fall Apart"
  • Sample Character Analysis Essay - "Dead Poet's Society"
  • Sample Compare and Contrast Essay - "Langston Hughes"
  • Sample Informative Essay - "Great War"
  • Sample Literary Devices Essay - "American Scholar"
  • Essay Tips: Syntax - What to Say About It
  • Sample Author Analysis Essay - "Edgar Allan Poe"
  • Sample Cause and Effect Essay - "Great Gatsby"
  • Sample Classification Essay - "Hamilton vs. Jefferson"
  • Essay Tips: 5 Ways to Improve Your Essay Scores
  • Violent Video Games are Not as Harmful as Parents Make Them Seem
  • Sample Letter to the Editor - "Animal Rights"
  • Morality: Fate Is Not In the Eye of the Beholder
  • 157,574 views (36 views per day)
  • Posted 12 years ago

Purdue Online Writing Lab Purdue OWL® College of Liberal Arts

APA Sample Paper

OWL logo

Welcome to the Purdue OWL

This page is brought to you by the OWL at Purdue University. When printing this page, you must include the entire legal notice.

Copyright ©1995-2018 by The Writing Lab & The OWL at Purdue and Purdue University. All rights reserved. This material may not be published, reproduced, broadcast, rewritten, or redistributed without permission. Use of this site constitutes acceptance of our terms and conditions of fair use.

Note:  This page reflects the latest version of the APA Publication Manual (i.e., APA 7), which released in October 2019. The equivalent resource for the older APA 6 style  can be found here .

Media Files: APA Sample Student Paper  ,  APA Sample Professional Paper

This resource is enhanced by Acrobat PDF files. Download the free Acrobat Reader

Note: The APA Publication Manual, 7 th Edition specifies different formatting conventions for student  and  professional  papers (i.e., papers written for credit in a course and papers intended for scholarly publication). These differences mostly extend to the title page and running head. Crucially, citation practices do not differ between the two styles of paper.

However, for your convenience, we have provided two versions of our APA 7 sample paper below: one in  student style and one in  professional  style.

Note: For accessibility purposes, we have used "Track Changes" to make comments along the margins of these samples. Those authored by [AF] denote explanations of formatting and [AWC] denote directions for writing and citing in APA 7. 

APA 7 Student Paper:

Apa 7 professional paper:.

AP English Language and Composition Exam Questions

Free-response questions and scoring information.

Download free-response questions from this year's exam and past exams along with scoring guidelines, sample responses from exam takers, and scoring distributions.

If you are using assistive technology and need help accessing these PDFs in another format, contact Services for Students with Disabilities at 212-713-8333 or by email at [email protected] .

2024: Free-Response Questions

Questions Scoring Samples and Commentary

  

 

  

  

 

-->

-->

2023: Free-Response Questions

QuestionsScoringSamples and Commentary

  

 

  

  

 

2022: Free-Response Questions

Questions

Scoring

Samples and Commentary

2021: Free-Response Questions

2021: Free-Response Questions

Questions

Scoring

Samples and Commentary

2020: Free-Response Questions

Note:  The table below features a selection of free-response questions and related scoring information from the 2020 exam. You can find all of the 2020 FRQs and corresponding scoring information in  AP Classroom .

2020 Exam: Free-Response Questions, Student Sample Responses, and Scoring Information

Prompts and Samples

Scoring Commentaries

Scoring Information







2019: Rescored Free-Response Questions

2019: Rescored Free-Response Questions and Updated Scoring Information
 SamplesCommentariesScoring Guidelines
Questions originally from the 2019 exam

2019: Free-Response Questions

2019: Free-Response Questions
QuestionsScoringSamples and Commentary

2018: Rescored Free-Response Questions

2018: Rescored Free-Response Questions and Updated Scoring Information
 SamplesCommentariesScoring Guidelines
Questions originally from the 2018 exam

2018: Free-Response Questions

2018: Free-Response Questions 
QuestionsScoringSamples and Commentary

2017: Free-Response Questions

2017: Free-Response Questions

Questions

Scoring

Samples and Commentary

2016: Free-Response Questions

2016: Free-Response Questions

Questions

Scoring

Samples and Commentary

2015: Free-Response Questions

2015: Free-Response Questions

Questions

Scoring

Samples and Commentary

2014: Free-Response Questions

2014: Free-Response Questions

Questions

Scoring

Samples and Commentary

2013: Free-Response Questions

2013: Free-Response Questions

Questions

Scoring

Samples and Commentary

2012: Free-Response Questions

2012: Free-Response Questions

Questions

Scoring

Samples and Commentary

2011: Free-Response Questions

2011: Free-Response Questions

Questions

Scoring

Samples and Commentary

2011: Form B

2011: Form B

Questions

Scoring

Samples and Commentary

2010: Free-Response Questions

2010: Free-Response Questions

Questions

Scoring

Samples and Commentary

2010: Form B

2010: Form B

Questions

Scoring

Samples and Commentary

2009: Free-Response Questions

2009: Free-Response Questions

Questions

Scoring

Samples and Commentary

 

2009: Form B

2009: Form B

Questions

Scoring

Samples and Commentary

 

2008: Free-Response Questions

2008: Free-Response Questions

Questions

Scoring

Samples and Commentary

2008: Form B

2008: Form B

Questions

Scoring

Samples and Commentary

2007: Free-Response Questions

2007: Free-Response Questions

Questions

Scoring

Samples and Commentary

2007: Form B

2007: Form B

Questions

Scoring

Samples and Commentary

2006: Free-Response Questions

2006: Free-Response Questions

Questions

Scoring

Samples and Commentary

2006: Form B

2006: Form B

Questions

Scoring

Samples and Commentary

2005: Free-Response Questions

2005: Free-Response Questions

Questions

Scoring

Samples

2005: Form B

2005: Form B

Questions

Scoring

Samples

2004: Free-Response Questions

2004: Free-Response Questions

Questions

Scoring

Samples

2004: Form B

2004: Form B

Questions

Scoring

Samples

2003: Free-Response Questions

2003: Free-Response Questions

Questions

Scoring

Samples

2003: Form B

2003: Form B

Questions

Scoring

Samples

2002: Free-Response Questions

2002: Free-Response Questions

Questions

Scoring

Samples

2002: Form B

2002: Form B

Questions

Scoring

Samples

2001: Free-Response Questions

2001: Free-Response Questions

Questions

Scoring

Samples

2000: Free-Response Questions

2000: Free-Response Questions

Questions

Scoring

Samples

1999: Free-Response Questions

1999: Free-Response Questions
Questions Scoring Samples

IMAGES

  1. How to Write a Research Paper in APA Format

    ap english research paper example

  2. AP Research

    ap english research paper example

  3. APA Basics: Fundamentals of Formatting Research Papers in APA Style

    ap english research paper example

  4. FREE 51+ Sample Paper Templates in MS Word

    ap english research paper example

  5. AP Lang Argument Essay Outline by Walden's Wonderland

    ap english research paper example

  6. AP English Language synthesis essay introduction

    ap english research paper example

VIDEO

  1. How Technology Has Affected Education?

  2. interview with Mentor for English Research Paper

  3. The Arrow Collar Man

  4. kivabe English Term Paper likhbo ||কিভাবে ইংরেজি টার্ম পেপার লিখব|| How to write English Term Paper

  5. Research Paper S6: موقع رائـع يساعدك في البحث Outline + Introduction (Undetectable--AI)

  6. how to make a dye sensitized solar cell HTL Braunau part2

COMMENTS

  1. AP Research Performance Task Sample and Scoring ...

    2016: Through-Course and End-of-Course Assessments. Download sample Academic Papers along with scoring guidelines and scoring distributions. If you are using assistive technology and need help accessing these PDFs in another format, contact Services for Students with Disabilities at 212-713-8333 or by email at [email protected].

  2. PDF AP® Research Academic Paper

    Some examples of. foods consumed. while abiding to the diet include fish, nuts, legumes, and eggs (Shi, El-Obeid, Li, Xu, Liu, 2019). According to the pre-existing research, the richness in natural foods plays a role in increasing.

  3. PDF AP® Research Academic Paper

    Sample: E Score: 3. This paper earned a score of 3. A method of content analysis is presented on page 4, followed by a description of the method on pages 4-5. The methods, however, are inconsistent, with two different descriptions given for how movies were chosen on pages 4 and 5.

  4. PDF AP Research Academic Paper

    AP. Research 2022 Scoring Commentary. Academic Paper. Overview. This performance task was intended to assess students' ability to conduct scholarly and responsible research and articulate an evidence-based argument that clearly communicates the conclusion, solution, or answer to their stated research question.

  5. PDF AP Research Academic Paper

    AP RESEARCH 2017 SCORING GUIDELINES Performance Task Rubric: Academic Paper NOTE: To receive the highest performance level presumes that the student also achieved the preceding performance levels in that row. ADDITIONAL SCORES: In addition to the scores represented on the rubric, readers can also assign scores of 0 (zero). - A score of . 0 . is assigned to a single row of the rubric when the ...

  6. PDF AP Research Academic Paper

    AP® RESEARCH 2017 SCORING GUIDELINES Performance Task Rubric: Academic Paper. The paper identifies a broad topic of inquiry The paper identifies a focused topic of inquiry and The paper explains the topic, purpose, and focus of the and/or a purpose. describes the purpose. inquiry and why further investigation of the topic is needed by ...

  7. PDF AP Research Academic Paper

    Score of 5. Rich Analysis of a New Understanding Addressing a Gap in the Research Base. Focuses a topic of inquiry with clear and narrow parameters, which are addressed through the method and the conclusion. Explicitly connects a topic of inquiry to relevant scholarly works of varying perspectives AND.

  8. AP English Sample Essays

    Use these sample AP English essays to get ideas for your own AP essays. These essays are examples of good AP-level writing. 1. Sample Definition Essay - "Success" ... Sample Research Paper - "The Patriot Act" Forty-five days after the September 11 terrorist attacks on the United States, Congress passed the USA PATRIOT Act, also known as the ...

  9. Sample Thesis Statements

    A thesis statement: tells the reader how you will interpret the significance of the subject matter under discussion. is a road map for the paper; in other words, it tells the reader what to expect from the rest of the paper. directly answers the question asked of you. A thesis is an interpretation of a question or subject, not the subject itself.

  10. PDF AP Research Academic Paper

    AP RESEARCH 2018 SCORING COMMENTARY . Academic Paper . Overview . This performance task was intended to assess students' ability to conduct scholarly and responsible research and articulate an evidence-based argument that clearly stated research question. More specifically, this performance task was intended to assess students' ability to:

  11. How to Write the AP Lang Rhetorical Analysis Essay (With Example)

    The AP Lang Rhetorical Analysis Essay is one of three essays included in the written portion of the AP English Exam. The full AP English Exam is 3 hours and 15 minutes long, with the first 60 minutes dedicated to multiple-choice questions. Once you complete the multiple-choice section, you move on to three equally weighted essays that ask you ...

  12. PDF AP® Research Academic Paper

    February in 2019, the NASDAQ Stock Market recorded an average of 12 million shares. traded daily (NasdaqTrader 2019). Based on a survey conducted in 2016, it is estimated that. 52% of Americans have money invested in the stock market (Jones and Saad 2016, 1).

  13. AP Research Assessment

    If you're using assistive technology and need help accessing the PDFs in this section in another format, contact Services for Students with Disabilities at 212-713-8333 or by email at [email protected]. For information about taking AP Exams, or other College Board assessments, with accommodations, visit the Services for Students with ...

  14. How to Write the AP Lang Argument Essay + Examples

    2. Pick one side of the argument, but acknowledge the other side. When you write the essay, it's best if you pick one side of the debate and stick with it for the entire essay. All your evidence should be in support of that one side. However, in your introductory paragraph, as you introduce the debate, be sure to mention any merit the ...

  15. AP English Language and Composition: Sample Rhetorical ...

    AP English Language and Composition: Sample Argument Question. The following paragraph is adapted from Mirror for Man, a book written by anthropologist Clyde Kluckhorn in the middle of the twentieth century. Read the passage carefully. Then, write an essay that examines the extent to which the author's characterization of the United States ...

  16. PDF AP Research Academic Paper

    Score of 1 Score of 2. Report on Existing Knowledge. Presents an overly broad topic of inquiry. Situates a topic of inquiry within a single perspective derived from scholarly works OR through a variety of perspectives derived from mostly non-scholarly works. Describes a search and report process.

  17. PDF AP Research Academic Paper

    AP® Research — Academic Paper 2021 Scoring Guidelines. The Response... Presents an overly broad topic of inquiry. Presents a topic of inquiry with narrowing scope or focus, that is NOT carried through either in the method or in the overall line of reasoning. overall line of reasoning, even though the focus or scope might still be narrowing.

  18. 170 AP Research Topics For Your Paper

    AP Seminar themes are the usual subject of discourse that often appear in any AP Seminar. These themes unite all the courses you study during your one year long course. You can consider these AP research topics: The ethics of algorithms on social media. Controversial supreme court ruling based on public opinion.

  19. Sample Research Paper

    Tragically, Congress drafted the Patriot Act and decreed that it would be the solution to America's problems. According to Congress, the Patriot Act would protect America from its enemies who operated on American soil. Many Americans unquestionably accepted the Act to avoid the risk of being labeled "unpatriotic.".

  20. PDF AP Research Academic Paper

    The balance of the paper consists of an extensive informal discussion of the topic focusing on the student's own suggestions for improving the performance of learning disabled students in the classroom. The paper did not score a 0 because it identifies a clear topic of investigation.

  21. APA Sample Paper

    Media Files: APA Sample Student Paper , APA Sample Professional Paper This resource is enhanced by Acrobat PDF files. Download the free Acrobat Reader. Note: The APA Publication Manual, 7 th Edition specifies different formatting conventions for student and professional papers (i.e., papers written for credit in a course and papers intended for scholarly publication).

  22. AP English Language and Composition Exam Questions

    Download free-response questions from this year's exam and past exams along with scoring guidelines, sample responses from exam takers, and scoring distributions. If you are using assistive technology and need help accessing these PDFs in another format, contact Services for Students with Disabilities at 212-713-8333 or by email at ssd@info ...

  23. PDF AP Research Academic Paper

    AP Research Academic Paper Sample Student Responses ... math,engineering,English (common classes ). You other choice could be sterling college where their acceptance rate is 44.6 and their fee is twenty one thousand which is a lot of money but still affordable.they have the same classes and